2014-2015/1er_STMG/DS/DST_0212/DST_0212.tex

470 lines
20 KiB
TeX

\documentclass[a4paper,12pt, table]{/media/documents/Cours/Prof/Enseignements/Archive/2014-2015/tools/style/classExamen}
\usepackage{/media/documents/Cours/Prof/Enseignements/Archive/2014-2015/2014_2015}
\usepackage{multicol}
% Title Page
\titre{}
% \seconde \premiereS \PSTMG \TSTMG
\classe{\PSTMG}
\date{12 février 2015}
\duree{2h}
\typedoc{Devoir commun}
\ptpres{3}
\printanswers
\begin{document}
\titlepage
\begin{questions}
\question[6]
Une entreprise fabriquant des paniers décide d'arrêter progressivement sa production: de 2300 paniers en janvier, elle diminue chaque mois la production de 200 paniers.
On note $u_n$ le nombre de panier produit le n-ième mois.
\begin{parts}
\part Expliquer pourquoi la suite est arithmétique. Donner la raison.
\begin{solution}
La suite est arithmétique pour passer de la production d'un mois donné au mois suivant, on enlève 200. La raison est -200.
\end{solution}
\part Calculer $u_1$ et $u_2$.
\begin{solution}
$u_0$ est la production en janvier avant le début de la diminution, donc $u_0 = 2300$.
\begin{eqnarray*}
u_1 & = & u_0 - 200 = 2300 - 200 = 2100 \\
u_2 & = & u_1 - 200 = 2100 - 200 = 1900 \\
\end{eqnarray*}
\end{solution}
\part Donner l'expression explicite de la suite $u_n$.
\begin{solution}
Expression explicite de $u_n$
\begin{eqnarray*}
u_n & = & u_0 + n\times r = 2300 + n\times (-200) = 2300 - 200n
\end{eqnarray*}
\end{solution}
\part Calculer $u_{11}$.
\begin{solution}
Calcul de $u_{11}$, on utilise la formule explicite.
\begin{eqnarray*}
u_{11} & = & 2300 - 200\times 11 = 100
\end{eqnarray*}
\end{solution}
\part À partir de quel mois la production s'arrêtera-t-elle?
\begin{solution}
On a vu qu'au bout de 11 mois, la production serait de 100 paniers (c'est $u_{11}$). Donc le mois d'après, la production s'arrêtera. AU bout de 12 mois, la production se sera arretée.
\end{solution}
\end{parts}
\question[9]
Une étude dand un centre hospitalier donne les résultats suivants:
\begin{itemize}
\item 1600 personnes travaillent dans ce centre.
\item 30\% sont des hommes.
\item 55\% des infirmières sont des femmes.
\item 1\% du personnel total sont des hommes aides-soignants.
\item 18\% du personnel sont des médecins.
\item Il n'y a pas de femmes chirurgien.
\end{itemize}
\begin{center}
\begin{tabular}{|c|*{5}{c|}}
\hline
& Aides-soignants & Infirmiers & Médecins & Chirurgiens & TOTAL \\
\hline
Femmes & 784 & & & & \\
\hline
Hommes & & & & & \\
\hline
TOTAL & & 440 & & & \\
\hline
\end{tabular}
\end{center}
\begin{parts}
\part Reproduire le tableau et le completer avec les données qui ne nécéssites pas de calculs.
\begin{solution}
En vert.
\begin{center}
\begin{tabular}{|c|*{5}{c|}}
\hline
& Aides-soignants & Infirmiers & Médecins & Chirurgiens & TOTAL \\
\hline
Femmes & 784 & \Ovalbox{1} = 242 & \Ovalbox{2} = 94 & \cellcolor{green} 0& \Ovalbox{3} = 1120 \\
\hline
Hommes & \Ovalbox{4} = 16 & \Ovalbox{5} = 198 & \Ovalbox{6} = 194 & \Ovalbox{7} = 72 & \Ovalbox{8} = 480 \\
\hline
TOTAL & \Ovalbox{9} = 800 & 440 & \Ovalbox{10} = 288 & \Ovalbox{11} = 72 & \cellcolor{green} 1600\\
\hline
\end{tabular}
\end{center}
Les numéros seront utilisés dans la question suivante.
\end{solution}
\part Finir de completer le tableau en justfiant chaque calculs.
\begin{solution}
\begin{itemize}
\item \Ovalbox{8}: $\frac{1600\times30}{100} = 480$
\item \Ovalbox{1}: $\frac{440\times55}{100} = 242$
\item \Ovalbox{4}: $\frac{1600\times 1}{100} = 16$.
\item \Ovalbox{10}: $\frac{1600 \times 18}{100} = 288$
\item \Ovalbox{3}: $1600 - 480 = 1120$
\item \Ovalbox{9}: $784 + 16 = 800$
\item \Ovalbox{2}: $1120 - 784 - 242 = 94$
\item \Ovalbox{5}: $440 - 242 = 198$
\item \Ovalbox{6}: $288 - 94 = 194$
\item \Ovalbox{11}: $1600 - 800 - 440 - 288 -16= 72$
\item \Ovalbox{7}: $480 - 16 - 198 - 194 = 72$
\end{itemize}
\end{solution}
\part Quelle est la proportion de femmes aide-soignantes dans ce centre?
\begin{solution}
Proportion de femmes aide-soignantes dans le centre:
\begin{eqnarray*}
\frac{784}{1600} & = & 0,49 = 49\%
\end{eqnarray*}
\end{solution}
\part Quelle est la proportion de médecins parmi les femmes?
\begin{solution}
Proportion de médecins parmi les femmes
\begin{eqnarray*}
\frac{94}{1120} & = & 0,084 = 8,4\%
\end{eqnarray*}
\end{solution}
\part Quelle est la proportion de femmes parmi les médecins?
\begin{solution}
Proportion de femmes parmi les médecins
\begin{eqnarray*}
\frac{94}{288} & = & 0,326 = 32,6\%
\end{eqnarray*}
\end{solution}
\end{parts}
\pagebreak
\question[6]
Un article coûtait 250\euro au premier janvier 2004. Il a subi une inflation (augmentation) de 4,6\% en 2004 puis de 3,8\% en 2005.
\begin{parts}
\part Calculer sont prix au premier janvier 2005 (après la première augmentation) et au premier janvier 2006 (après la deuxième augmentation).
\begin{solution}
Prix en janvier 2005 (après une augmentation de 4,6\%)
\begin{eqnarray*}
250\times(1+\frac{4,6}{100}) & = & 261,5
\end{eqnarray*}
Prix en janvier 2006 (après l'augmentation de 3,8\%)
\begin{eqnarray*}
261,5\times(1 + \frac{3,8}{100}) & \approx & 271,4
\end{eqnarray*}
\end{solution}
\part Calculer la variation absolue entre le premier janvier 2004 et et le premier janvier 2006.
\begin{solution}
Variation absolue entre janvier 2004 et janvier 2006
\begin{eqnarray*}
y_2 - y_1 & = & 271,4 - 250 = 21,4
\end{eqnarray*}
\end{solution}
\part Calculer la variation relative entre le premier janvier 2004 et et le premier janvier 2006.
\begin{solution}
Variation relative entre janvier 2004 et janvier 2006
\begin{eqnarray*}
\frac{y_2 - y_1}{y_1} & = & \frac{271,4 - 250}{250} = 0,0856 = 8,56\%
\end{eqnarray*}
\end{solution}
\part Quelle inflation faudrait-il en 2006 pour que son prix atteigne 300\euro?
\begin{solution}
Si on veut atteindre 300\euro en 2006, il faudrai
\begin{eqnarray*}
\frac{y_2 - y_1}{y_1} & = & \frac{300 - 271,4}{271,4} = 0,105 = 10,5\%
\end{eqnarray*}
\end{solution}
\end{parts}
\question[10]
% Depuis repère 67 p 157
Un magasin a annoncé sa journée de promotion par une distribution de tracts sur lesquels était indiqué:
\begin{center}
\textit{Grande journée de promotion! Dépensez moins!}
\end{center}
\textbf{Partie 1}\\
Le tableau ci-dessous donne les montants en euros, arrondis à l'unité, des achats effectués par les 80 clients du magasin pendant une journée ordinaire.
\definecolor{lightgray}{gray}{0.9}
\rowcolors{1}{lightgray}{}
\begin{center}
\begin{tabular}{|*{8}{c|}}
\hline
2 &3 &5 &5 &5 &8 &8 &8\\
\hline
8 &10 &10 &10 &10 &10 &10 &10\\
\hline
11 &13 &14 &14 &14 &20 &20 &20\\
\hline
20 &20 &20 &21 &24 &24 &25 &26\\
\hline
30 &30 &30 &30 &30 &30 &31 &33\\
\hline
33 &35 &36 &38 &38 &38 &38 &38\\
\hline
39 &39 &39 &39 &39 &40 &40 &40\\
\hline
40 &40 &40 &40 &40 &40 &42 &42\\
\hline
42 &43 &43 &43 &44 &44 &45 &45\\
\hline
45 &45 &45 &46 &46 &47 &55 &60\\
\hline
\end{tabular}
\end{center}
\rowcolors{1}{}{}
\begin{parts}
%1pts
\part Tracer le tableau des effectifs de cette série statistique.
\begin{solution}
Tableau des effectifs
\begin{center}
\begin{tabular}{|r|*{29}{c|}}
\hline
Montants & 2 & 3 & 5 & 8 & 10 & 11 & 13 & 14 & 20 & 21 & 24 & 25 & 26 & 30 & 31 & 33 & 35 & 36 & 38 & 39 & 40 & 42 & 43 & 44 & 45 & 46 & 47 & 55 & 60 \\
\hline
Effectifs & 1 & 1 & 3 & 4 & 7 & 1 & 1 & 3 & 6 & 1 & 2 & 1 & 1 & 6 & 1 & 2 & 1 & 1 & 5 & 5 & 9 & 3 & 3 & 2 & 5 & 2 & 1 & 1 & 1 \\
\hline
\end{tabular}
\end{center}
\end{solution}
% J'aimerai qu'ils aient à tracer un histogramme ici!
\part
\begin{subparts}
%1pt
\subpart Déterminer le pourcentage de clients ayant effectué des achats pour un montant ne dépassant pas les 27\euro.
\begin{solution}
Pourcentage des clients ayant fait des achats pour moins de 27\euro
\begin{eqnarray*}
\frac{32}{80} & = & 0,4 = 40\%
\end{eqnarray*}
\end{solution}
%1pt
\subpart Déterminer le pourcentage de clients ayant effectué des achats entre 30\euro\; et 40\euro\; inclus.
\begin{solution}
Pourcentage des clients ayant fait des achats entre 30 et 40\euro.
\begin{eqnarray*}
\frac{30}{80} & = & 0,375 = 37,5\%
\end{eqnarray*}
\end{solution}
\end{subparts}
% 2pt
\part Calculer la moyenne de cette série statistique.
\begin{solution}
Calcul de la moyenne de cette série
\begin{eqnarray*}
\bar{x} & = & \frac{2 + 3 + 5\times 3 + 8 \times 4 + \cdots + 55 + 60}{80} = 29,29
\end{eqnarray*}
\end{solution}
\part
\begin{subparts}
%1pt
\subpart Déterminer le minimum et le maximum de cette série statistique.
\begin{solution}
En lisant le tableau de valeurs:
\begin{itemize}
\item Mininum: 2
\item Maximum: 60
\end{itemize}
\end{solution}
%2pts
\subpart Déterminer la médiane de cette série statistique.
\begin{solution}
Médiane de cette série. Dans le sujet les données sont déjà rangées par ordre croissant.
Effectif total: 80
Position de la médiane: $\frac{80}{2} = 40$ Donc la médiane se trouver entre la 40 et la 41ième valeur. Donc $Me =33 $.
\end{solution}
%2pts
\subpart Déterminer les quartiles de cette série statistique.
\begin{solution}
Position du premier quartile: $\frac{1}{4} \times 80 = 20$. Donc le premier quartile se trouve entre la 20ième et la 21ième valeur. Donc $Q_1 = 14$.
Position du troisième quartile: $\frac{3}{4} \times 80 = 60$. Donc le troisième quartile se trouve entre la 60ième et la 61ième valeur. Donc $Q_3 = 40$.
\end{solution}
\end{subparts}
\begin{EnvFullwidth}
\textbf{Partie 2}\\
Un étude similaire a été faite sur 80 clients lors d'une journée de promotion. Cette étude a donné le diagramme en boite suivant:
\begin{center}
\begin{tikzpicture}[xscale = 0.1]
\tkzInit[xmin=0,xmax=80,xstep=10]
\boxplot{1}{5}{45}{55}{63}{75}
\foreach \x in {0,10,...,90} \draw(\x,0)node[rotate=90] {$-$} node[below]{\x};
\draw[->] (0,0) -- (95,0);
\end{tikzpicture}
\end{center}
\end{EnvFullwidth}
%1pts
\part Quels sont les 5 informations que l'on peut lire sur ce diagramme à propos des dépenses des clients lors d'une journée de promotion?
\begin{solution}
Sur le diagramme en boite, on peut lire:
\begin{center}
Min = 5 \hfill $Q_1 = 45$ \hfill $Me = 55$, \hfill $Q_3 = 65$ \hfill Max = 75
\end{center}
\end{solution}
%1pts
\part Tracer l'un au dessus de l'autre les diagrammes en boites d'une journée de promotion et d'une journée ordinaire.
\begin{solution}
\begin{center}
\begin{tikzpicture}[xscale = 0.1]
\tkzInit[xmin=0,xmax=80,xstep=10]
\boxplot{1}{5}{45}{55}{63}{75}
\boxplot{3}{2}{14}{33}{40}{60}
\foreach \x in {0,10,...,90} \draw(\x,0)node[rotate=90] {$-$} node[below]{\x};
\draw[->] (0,0) -- (95,0);
\end{tikzpicture}
\end{center}
\end{solution}
%1pts
\part En comparant ces deux diagrammes en boite, commenter l'annonce du magasin.
\begin{solution}
On remarque que lors d'une journée ordinaire, 75\% des clients depensent moins de 40\euro alors que lors d'une journée de promotions, 75\% depensent plus de 45\euro. L'annonce du magasin donc fausse, les clients ne dépensent pas moins un jour de promotion.
\end{solution}
\end{parts}
\question[6]
\begin{itshape}
Cet exercice est un questionnaire à choix multiplies (QCM).
Pour chaque question, une seule des 4 réponses proposées est correcte.
Indiquer sur la copie le numéro de la question ainsi que la réponse choisie. Aucune justification n'est demandée.
Une réponse juste rapporte 1,5~point, une réponse fausse ou l'absence de réponse ne rapporte ni n'enlève de point. Si le total des points est négatif, la note attribuée à l'exercice est ramenée à 0.
\end{itshape}
\begin{parts}
\part Combien l'équation $-6x + x^2 + 9 = 0$ a-t-elle de solution?
\begin{multicols}{2}
\begin{subparts}
\subpart Aucune solution
\subpart Une solution
\subpart deux solutions
\subpart Une autre réponse
\end{subparts}
\end{multicols}
\begin{solution}
Pour connaître le nombre de solution d'une équation du 2nd degré, il faut calculer le discriminant ($\Delta$). Attention ici les coefficients n'étaient pas rangé comme il le faut. Ici $a = 1$, $b = -6$ et $c = 9$.
\begin{eqnarray*}
\Delta & = & b^2 - 4ac = (-6)^2 - 4\times 1 \times 9 = 36 - 36 = 0
\end{eqnarray*}
L'équation a donc une seule solution.
La bonne réponse est donc la (b)
\end{solution}
\part Quelle formule doit-t-on taper dans \texttt{B2}, puis recopié vers le bas pour completer le tableau
\includegraphics[scale=0.4]{./fig/tab_qcm}
\begin{subparts}
\subpart \Ovalbox{\texttt{=2*0*0-2*0+2}}
\subpart \Ovalbox{\texttt{=2*A1*A1-2*A1+2}}
\subpart \Ovalbox{\texttt{=2*B2*B2-2*B2+2}}
\subpart Une autre réponse
\end{subparts}
\begin{solution}
La première solution n'est pas valable car même si ce calcul donnerai le bon résultat pour la première valeur, on ne pourrai pas étirer la formule vers le bas.
La deuxième solution n'est pas non plus valable, car dans la case \texttt{A1} il y a la valeur \texttt{x} ce qui ne permet pas de calculer l'image de 0 par $f$.
La troisième solution ne marche pas non plus, car la case \texttt{B2} est la case où on veut que le résultat soit affiché.
La bonne formule aurait été \texttt{=2*A2*A2-2*A2+2}. Donc la bonne réponse est (d).
\end{solution}
\part Le tableau de signe de la fonction $f(x) = x^2 - 6x + 9$ est
\begin{subparts}
\subpart \begin{tikzpicture}[scale = 0.7]
\tkzTabInit[]{$x$/1,$f(x)$/1}{$-\infty$, $+\infty$}
\tkzTabLine{,+,}
\end{tikzpicture}
\subpart \begin{tikzpicture}[scale = 0.7]
\tkzTabInit[]{$x$/1,$f(x)$/1}{$-\infty$,3, $+\infty$}
\tkzTabLine{, +, z , +,}
\end{tikzpicture}
\subpart \begin{tikzpicture}[scale = 0.7]
\tkzTabInit[]{$x$/1,$f(x)$/1}{$-\infty$,-3, 3, $+\infty$}
\tkzTabLine{, +, z, -, z, +,}
\end{tikzpicture}
\subpart Une autre réponse
\end{subparts}
\begin{solution}
Pour tracer le tableau de signe d'un polynôme du 2nd degré, il faut commencer par calculer le discriminant (ce calcul a déjà été fait pour la première question du QCM)
\begin{eqnarray*}
\Delta & = & 0
\end{eqnarray*}
Il y a donc une seule racine.
\begin{eqnarray*}
x_1 & = & \frac{-b}{2a} = \frac{6}{2\times 1} = 3
\end{eqnarray*}
De plus ici $a = 1 > 0$ sont ce polynôme est toujours positif. La bonne réponse est donc la (b).
\end{solution}
\part Le graphique de la fonction $f(x) = -3x^2 + 3x + 1$ est
\begin{multicols}{2}
\begin{subparts}
\subpart \begin{tikzpicture}[baseline=(O.base),start chain, scale=0.5]
\repereNoGrid{-4}{4}{-4}{4}
\clip (-4,-4) rectangle (4,4);
\draw[very thick, domain=-4:4, color=red] plot [samples=100] (\x, {-3*\x*\x + 3*\x + 1});
\end{tikzpicture}
\subpart \begin{tikzpicture}[baseline=(O.base),start chain, scale=0.5]
\repereNoGrid{-4}{4}{-4}{4}
\clip (-4,-4) rectangle (4,4);
\draw[very thick, domain=-4:4, color=red] plot [samples=100] (\x, {3*\x*\x + 3*\x + 1});
\end{tikzpicture}
\subpart \begin{tikzpicture}[baseline=(O.base),start chain, scale=0.5]
\repereNoGrid{-4}{4}{-4}{4}
\clip (-4,-4) rectangle (4,4);
\draw[very thick, domain=-4:4, color=red] plot [samples=100] (\x, {3*\x + 1});
\end{tikzpicture}
\subpart Une autre réponse
\end{subparts}
\end{multicols}
\begin{solution}
On peut commencer par éliminer la réponse (c) car c'est la courbe représentative d'une fonction affine et là $f$ est un polynôme du 2nd degré.
Ici $a = -3 < 0$ donc les branches de la parabole doivent être vers le bas ce qui élimine la réponse (b).
On peut vérifier que la réponse (a) correspond à la fonction $f$. Pour cela on peut tracer le graphique de cette fonction sur la calculatrice ou faire l'étude du polynôme avec le discriminant.
\end{solution}
\end{parts}
\end{questions}
\end{document}
%%% Local Variables:
%%% mode: latex
%%% TeX-master: "master"
%%% End: